LSAT and Law School Admissions Forum

Get expert LSAT preparation and law school admissions advice from PowerScore Test Preparation.

 Administrator
PowerScore Staff
  • PowerScore Staff
  • Posts: 8916
  • Joined: Feb 02, 2011
|
#72533
Complete Question Explanation


Weaken-SN. The correct answer choice is (E)

This problem is a complete conditional argument containing conditional
premises and a conditional conclusion. Here is a breakdown of the
argument:

..... HT = nations that place a high tax on income
..... NI = negative incentive for technological innovation
..... FB = fall behind in the international arms race; also, wind up in a
..... strategically disadvantageous position
..... LV = lose voice in world affairs

The first sentence contains two sufficient condition indicators (the word
“all”) and can be diagrammed as a chain:

..... HT :arrow: NI :arrow: FB

The next sentence paraphrases “fall behind in the international arms
race” as “wind up in a strategically disadvantageous position” and can be
diagrammed as:

..... FB :arrow: LV

Because the two statements have FB in common, a single long chain can
be created:

..... HT :arrow: NI :arrow: FB :arrow: LV

From our discussion of conditional reasoning we know that a chain of this
length contains many inferences. The conclusion, when paraphrased, tries
to make a contrapositive:

..... ..... The phrase “nation wants to maintain its value system and way of
..... ..... life” is a very rough equivalent of “not wind up in a strategically
..... ..... disadvantageous position” and “not lose a voice in world affairs.”
..... ..... The paraphrase is not a perfect equivalent because the conclusion
..... ..... discusses values, and the premises do not. For our purposes, we
..... ..... will symbolize this condition as:

..... ..... ..... FB
..... ..... ..... and
..... ..... ..... LV

The phrase “must not allow its highest tax bracket to exceed 30
percent of income” is the equivalent of HT. Thus, the diagram for
the conclusion is:

..... ..... ..... FB
..... ..... ..... and :arrow: HT30
..... ..... ..... LV

Thus, based on the chain of reasoning provided, we have a reasonable
conclusion, but not a perfect one because the paraphrase was not exact.
The question stem is a WeakenX, which means that four of the answers
will weaken the argument and the one correct answer will either have no
effect on the argument or will strengthen the argument.

Answer choice (A): This answer attacks the necessary condition of the
conclusion by showing that taxes could exceed 30% before problems
occurred.

Answer choice (B): This answer attacks the first half of the first sentence,
which states that high taxes necessarily produce a negative incentive for
technological innovation. Because taxes lower an individual’s income,
the higher the tax, the greater the relative restriction on making money.
Answer choice (B) shows that higher taxes would not necessarily produce
low innovation because innovators do not care about the amount they earn.

Answer choice (C): This answer attacks the part of the argument that
equates “fall behind in the international arms race” as “wind up in a
strategically disadvantageous position.” If the two are not equated, then the
chain of premises breaks down.

Answer choice (D): Like (C), this attacks a portion of the argument where
the author equates terms. In this case, the paraphrase in the conclusion was
not exact, and this answer exploits that gap.

Answer choice (E): This is the correct answer. The answer does not hurt
the argument because the stimulus specifically states that “Those nations
that, through historical accident or the foolishness of their political
leadership, wind up in a strategically disadvantageous position....” So,
the actual reason the nation ends up in a disadvantageous position is not
critical. It could be either foolishness or historical accident. Thus, an
answer that asserts that it is foolishness and not historical accident has no
effect on the argument.
 ksandberg
  • Posts: 21
  • Joined: Sep 03, 2016
|
#37900
Hello,

I was between B and E for this question and although I chose E, I am not completely understanding why B is incorrect. The argument made by the politician is that heavily taxing people hampers technological innovation, which leads to a chain of negative consequences and therefore we shouldn’t tax people too much. B makes sense as an incorrect answer choice because if it turns out that what people make, chain link A, does not impact technological innovation, chain link B, then the question is weakened. However, to me this answer choice seems out of scope and thus is not one that I would say truly weakens the question. At no point does the politician say that the reason high income tax is bad is because it prevents people from making a great deal of money. Isn't the concept of a great deal of money subjective anyway? You can still make a great deal of money while living in a socialist country. Can you please provide some clarification on this?
 AthenaDalton
PowerScore Staff
  • PowerScore Staff
  • Posts: 296
  • Joined: May 02, 2017
|
#37949
Great question!

The politician's argument is that taxing income :arrow: negative incentives for technological innovation.

Essentially, he is arguing that if inventors are heavily taxed and cannot make a lot of money from their inventions, they will produce fewer innovations. A real-world application of this argument would be, "If Steve Jobs had to pay a 90 percent income tax, he wouldn't have invented the iPhone because he wouldn't have profited much from his invention."

Answer choice (B) tells us that making a great deal of money is an insignificant factor driving technological innovation.

If the statement in (B) is true, it undermines the key link in the politician's argument between making money and inventing things. The real-world application of this idea would be, "Steve Jobs' love of technology drove him to invent the iPhone. He loved inventing things so much he would have done it for free."

You are right that the phrase "a great deal of money" is relative, but it still weakens the argument a little bit and is therefore not the right answer.

An inventor subject to a 90 percent income tax would certainly make less money than an inventor subject to a 30 percent tax, so if the politician's argument were correct (that inventors create things in order to make money), the higher tax would decrease innovation. Answer choice (B) tells us that profit is not a significant factor driving invention, so the 90 percent tax vs 30 percent tax wouldn't impact the production of new technologies.

I hope this clears things up. Good luck studying!
 ksandberg
  • Posts: 21
  • Joined: Sep 03, 2016
|
#38192
Thank you so much for the real world example. That definitely helped.
 Jerrymakehabit
  • Posts: 52
  • Joined: Jan 28, 2019
|
#62250
Dear Teacher,

I am reading the Bible and it explains choice A as "This answer attacks the necessary condition of the conclusion by showing that taxes could exceed 30% before problems occurred"

The argument tells that taxes exceeding 30% :arrow: negative incentive innovations. And, incentive innovation :arrow: taxes can not exceed 30%. Above are my understanding of the argument.

Choice A says negative incentive innovations :arrow: taxes exceeds 45%. It is true that choice A attacks the necessary condition "can not exceed 30%" by using "taxes exceeds 45%", but the sufficient condition is different here. In argument it is "incentive innovation" but in choice A it is "negative incentive innovations". I thought you can only attack the necessary condition when keeping the sufficient condition the same. Am I wrong? Or, what is wrong with my logic here?

Also, for choice B, can I interpret the choice as it is possible that high tax :arrow: incentive innovation?

I would really appreciate it if you could help point out the flaws in my logic here!
 Malila Robinson
PowerScore Staff
  • PowerScore Staff
  • Posts: 296
  • Joined: Feb 01, 2018
|
#62260
Hi Jerrymakehabit,
It looks like your diagramming is correct in relation to Answer A. What may be throwing you off may be a reliance on the diagrams without focusing what they are really saying. For answer A the relevant part of the stimulus is that 30% is the cutoff for avoiding problems with technological innovation. And Answer A says no, it's not 30% it's 45%, so it weakens the argument by raising the cutoff for avoiding problems with technological innovations.

I am not quite seeing how you reached your diagram for Answer B, can you explain it a little more fully?
Hope that helped!
-Malila
 Jerrymakehabit
  • Posts: 52
  • Joined: Jan 28, 2019
|
#62297
Malila Robinson wrote:Hi Jerrymakehabit,
It looks like your diagramming is correct in relation to Answer A. What may be throwing you off may be a reliance on the diagrams without focusing what they are really saying. For answer A the relevant part of the stimulus is that 30% is the cutoff for avoiding problems with technological innovation. And Answer A says no, it's not 30% it's 45%, so it weakens the argument by raising the cutoff for avoiding problems with technological innovations.

I am not quite seeing how you reached your diagram for Answer B, can you explain it a little more fully?
Hope that helped!
-Malila
Hi Malila,

Thank you for the explain!

I just begin to learn LSAT with the Bible (one month so far) and I do find I have the tendency to largely reply on diagrams. Maybe it is because I have been trained as an engineer. Diagrams are like equations for me. It is tricky that sometimes have to stick to diagrams but sometimes have to understand what they are intending to say. But I will try to change my mindset to LSAT mode by keep practicing.

For Answer B, "Making a great deal of money is an insignificant factor" means innovators could accept high tax and then they still make innovations. So I draw it high tax :arrow: incentive innovation which weakens the high tax :arrow: negative incentive innovation in the argument. Does it make sense? May I know how you would interpret the choice?

Thanks
Jerry
 Rachael Wilkenfeld
PowerScore Staff
  • PowerScore Staff
  • Posts: 1358
  • Joined: Dec 15, 2011
|
#62319
Hi Jerry,

I think this might make more sense if we take it from the stimulus. Part of the trouble here is that there are so many conditionals, and it can make it tricky to sort out what to connect and how.

The first conditional is right at the start:

high income tax :arrow: negative incentive for innovation

The author then moves on to talk about something similar, countries where technological innovation is hampered:

innovation hampered :arrow: fall behind in international arms race

The next conditional makes another slight shift, from failing behind in the arms race to just generally strategically disadvantageous position:

strategically disadvantageous position :arrow: lose voice in world affairs

Finally, the author draws a conclusion that seems like a pretty big logical leap:

Maintain values/way of life :arrow: highest tax bracket must not exceed 30%

One of the really difficult things about this stimulus is that the conditionals ALMOST connect. But the necessary conditions don't quite match the next sufficient condition. That gives us a lot of room to attack. The author WANTS us to make the assumption, for example, that a falling behind in the international arms race is equivalent to being in a strategically disadvantageous position. We don't know that's the case though. Answer choice (C) addresses this by directly attacking that link.

So let's turn now to your question. It seems like you are trying to make things conditional that are not conditional. The answer choices here aren't conditional, the stimulus is. By trying to make the answer choices conditional, you are misunderstanding the structure of the answers, and how they impact the stimulus. It can be so tempting to make everything conditional. Once you realize how powerful conditional reasoning is, you want it to be everywhere! But forcing non-conditional statements into a conditional structure is a mistake that is just as dangerous as a mistaken reversal or mistaken negation.

In answer choice (B), the answer attacks the link between high taxes and technological innovation being hampered. We don't attack the first conditional. We accept that high taxes will result in negative incentive for technological innovation. But the answer choice attacks the idea that the negative incentives are enough to get to the next conditional They aren't enough to actually hamper technological innovation, because even though there is a negative incentive, it's not one that the innovators really care about. If money is an insignificant factor, then the taxes aren't going to hamper innovation. It hurts the link between the first and second conditionals.

Hope that helps!
Rachael
 Jerrymakehabit
  • Posts: 52
  • Joined: Jan 28, 2019
|
#62327
Rachael Wilkenfeld wrote:Hi Jerry,

I think this might make more sense if we take it from the stimulus. Part of the trouble here is that there are so many conditionals, and it can make it tricky to sort out what to connect and how.

The first conditional is right at the start:

high income tax :arrow: negative incentive for innovation

The author then moves on to talk about something similar, countries where technological innovation is hampered:

innovation hampered :arrow: fall behind in international arms race

The next conditional makes another slight shift, from failing behind in the arms race to just generally strategically disadvantageous position:

strategically disadvantageous position :arrow: lose voice in world affairs

Finally, the author draws a conclusion that seems like a pretty big logical leap:

Maintain values/way of life :arrow: highest tax bracket must not exceed 30%

One of the really difficult things about this stimulus is that the conditionals ALMOST connect. But the necessary conditions don't quite match the next sufficient condition. That gives us a lot of room to attack. The author WANTS us to make the assumption, for example, that a falling behind in the international arms race is equivalent to being in a strategically disadvantageous position. We don't know that's the case though. Answer choice (C) addresses this by directly attacking that link.

So let's turn now to your question. It seems like you are trying to make things conditional that are not conditional. The answer choices here aren't conditional, the stimulus is. By trying to make the answer choices conditional, you are misunderstanding the structure of the answers, and how they impact the stimulus. It can be so tempting to make everything conditional. Once you realize how powerful conditional reasoning is, you want it to be everywhere! But forcing non-conditional statements into a conditional structure is a mistake that is just as dangerous as a mistaken reversal or mistaken negation.

In answer choice (B), the answer attacks the link between high taxes and technological innovation being hampered. We don't attack the first conditional. We accept that high taxes will result in negative incentive for technological innovation. But the answer choice attacks the idea that the negative incentives are enough to get to the next conditional They aren't enough to actually hamper technological innovation, because even though there is a negative incentive, it's not one that the innovators really care about. If money is an insignificant factor, then the taxes aren't going to hamper innovation. It hurts the link between the first and second conditionals.

Hope that helps!
Rachael
Hi Rachael,

Thank you for the detailed explanation. I learned from your explanation that "Finally, the author draws a conclusion that seems like a pretty big logical leap". The author tried to lead us to believe in this big logic loop but Choice C breaks the author's intention to lead us in believing this.

Choice A, I learned from the Bible that "must" introduce a necessary condition, which is why I immediately drew negative incentive innovations :arrow: taxes exceeds 45%. As a rule of thumb in weakening conditional reasoning questions, we can only attack the necessary condition when keeping the sufficient condition same. Is that correct?

Choice B, based on your explanation, B attacks the invisible link negative incentive for innovation :arrow: innovation hampered.

Thank you!
Jerry
 Adam Tyson
PowerScore Staff
  • PowerScore Staff
  • Posts: 5153
  • Joined: Apr 14, 2011
|
#62343
Hey there Jerry, let me see if I can clarify (and I think you have got it already). The way to weaken a conditional claim is to show that the necessary condition is not, in fact, necessary. That is, it is possible for the sufficient condition to occur and the necessary condition to not occur. In other words, the sufficient condition is NOT sufficient after all, because the necessary condition isn't actually required.

I would say you are also right about answer B here: there may be a negative incentive for innovation, but if the negative incentive is very small then it might not actually hamper it.

Looks like you've got it!

Get the most out of your LSAT Prep Plus subscription.

Analyze and track your performance with our Testing and Analytics Package.